Ventisei

Il forum di Base5, dove è possibile postare problemi, quiz, indovinelli, rompicapo, enigmi e quant'altro riguardi la matematica ricreativa e oltre.

Moderatori: Gianfranco, Bruno

peppe
Livello 7
Livello 7
Messaggi: 906
Iscritto il: gio mag 26, 2005 2:41 pm
Località: Cirò Marina KR

Ventisei

Messaggio da peppe »

Fra le varie proprietà (anzi curiosità) del numero 26 quella che più di tutte mi incuriosisce è questa:

È l'unico numero esistente interposto fra un quadrato (25 = 5^2) e un cubo (27 = 3^3).

Sarei curioso di sapere come fece,"il principe dei dilettanti" , Pierre de Fermat a dimostrarlo.
Qui ho trovato una spiegazione che, per me, è arabo!!

Ringrazio anticipatamente chi è in grado di fornirmi una spiegazione comprensibile. Grazie.
---
P.s.
E a proposito del numero 26, di cubi e quadrati:
Se consideriamo:
- la differenza tra la maggiore delle due cifre (dello stesso numero 26) e la minore, si ottiene:

6 – 2 = 4 = 2^2
- la somma delle due stesse cifre, si ottiene:
6 + 2 = 8 = 2^3
si trovano due potenze consecutive di uguale base.

Annarita Ruberto sul suo blog segnala un intervento di
Maria Intagliata
dal titolo: "un giochino matematico con il 26"
Peppe

peppe
Livello 7
Livello 7
Messaggi: 906
Iscritto il: gio mag 26, 2005 2:41 pm
Località: Cirò Marina KR

Re: Ventisei

Messaggio da peppe »

Sto quasi dimenticando il codice $\LaTeX$
La pagina che Pietro gli ha dedicato contiene numerose funzioni. In questo topic di poco interesse
provo a postarne qualche altra copiata dal forum.
Racchiudo il codice $\LaTeX$fra i tag (code) e (/code) [fra parentesi quadre] in modo
che non venga visualizzata la funzione e subito dopo li tolgo per visualizzarla:

Codice: Seleziona tutto

[tex] a \dot {=} b [/tex]
$a \dot {=} b$
+++

Codice: Seleziona tutto

[tex]C = {{8m + 2k} \choose {2k}}[/tex]
$C = {{8m + 2k} \choose {2k}}$
+++

Codice: Seleziona tutto

[tex](AQ)^2=(AH_{2})^2+(H_{2}Q)^2[/tex]
$(AQ)^2=(AH_{2})^2+(H_{2}Q)^2$
+++

Codice: Seleziona tutto

[tex](AQ)^2=(\frac{\sqr{3}}{2}L)^2+(\frac{L}{6})^2[/tex]
$(AQ)^2=(\frac{\sqr{3}}{2}L)^2+(\frac{L}{6})^2$
+++

Codice: Seleziona tutto

[tex] (AP)^2=(AE)^2+(EP)^2-2(AE)(EP)cos(60^\circ)[/tex
$(AP)^2=(AE)^2+(EP)^2-2(AE)(EP)cos(60^\circ)$
+++

Codice: Seleziona tutto

[tex] \frac{\sqr{7}}{3}L[/tex]= [tex]\frac{1}{\sqr{7}}L+\frac{1}{3\cdot\sqr{7}}L+L_{1}[/tex]
$\frac{\sqr{7}}{3}L$= $\frac{1}{\sqr{7}}L+\frac{1}{3\cdot\sqr{7}}L+L_{1}$
+++

Codice: Seleziona tutto

[tex]A\equiv(0,\ 0)[/tex] 
$A\equiv(0,\ 0)$
+++

Codice: Seleziona tutto

[tex]C\equiv(\frac{1}{2},\ \frac{\sqr3}{2})[/tex] 
$C\equiv(\frac{1}{2},\ \frac{\sqr3}{2})$
+++
Peppe

peppe
Livello 7
Livello 7
Messaggi: 906
Iscritto il: gio mag 26, 2005 2:41 pm
Località: Cirò Marina KR

Re: Ventisei

Messaggio da peppe »

Codice: Seleziona tutto

[tex]\displaystyle \frac{shih}{a - a_p} = \frac{ab_p + a_pb}{a - a_p} = \frac{210}{3} = 70[/tex]
$\displaystyle \frac{shih}{a - a_p} = \frac{ab_p + a_pb}{a - a_p} = \frac{210}{3} = 70$
+++

Codice: Seleziona tutto

[tex]\sqr{2}^{\sqr{2}^{\sqr{2}^{\sqr{2}^{...}}}}[/tex]
$\sqr{2}^{\sqr{2}^{\sqr{2}^{\sqr{2}^{...}}}}$
+++

Codice: Seleziona tutto

[tex]3^{(3^3)}[/tex]
$3^{(3^3)}$
+++

Codice: Seleziona tutto

[tex]x^{x^{x^{x^{...}}}}[/tex] 
$x^{x^{x^{x^{...}}}}$
+++

Codice: Seleziona tutto

[tex]\sqr{x+\sqr{x+\sqr{x+\sqr{x+...}}}}[/tex]
$\sqr{x+\sqr{x+\sqr{x+\sqr{x+...}}}}$
+++

Codice: Seleziona tutto

[tex]P=k[/tex]  con [tex] y \in[10^{k}\ , 10^{k-1}][/tex]
$P=k$ con $y \in[10^{k}\ , 10^{k-1}]$
+++

Codice: Seleziona tutto

[tex]y \in[10^{k-1}\,10^{k}][/tex]
$y \in[10^{k-1}\,10^{k}]$
+++

Codice: Seleziona tutto

[tex] x^{3/7}k^{4}z^{3}\leq21 [/tex]
$x^{3/7}k^{4}z^{3}\leq21$
+++

Codice: Seleziona tutto

[tex] \log_{e}x\geq6 [/tex]
$\log_{e}x\geq6$
+++

Codice: Seleziona tutto

[tex] \sqr{\sqr{x+z}} [/tex]
$\sqr{\sqr{x+z}}$
+++

Codice: Seleziona tutto

[tex] x=\frac{-b\pm\sqrt{b^2-4ac}}{2a} [/tex]
$x=\frac{-b\pm\sqrt{b^2-4ac}}{2a}$
+++

Codice: Seleziona tutto

[tex] \fs{5} x=\frac{-b\pm\sqrt{b^2-4ac}}{2a} [/tex]
$\fs{5} x=\frac{-b\pm\sqrt{b^2-4ac}}{2a}$
+++
Peppe

peppe
Livello 7
Livello 7
Messaggi: 906
Iscritto il: gio mag 26, 2005 2:41 pm
Località: Cirò Marina KR

Re: Ventisei

Messaggio da peppe »

Codice: Seleziona tutto

[tex]1+m+n\cdot\sqr{3} = [2+\sqr{3}]^{2r-1}[/tex]
$1+m+n\cdot\sqr{3} = [2+\sqr{3}]^{2r-1}$
+++

Codice: Seleziona tutto

[tex] x=\sqr[3]{-4+i\cdot\sqr{11}}+\sqr[3]{-4-i\cdot\sqr{11}} [/tex]
$x=\sqr[3]{-4+i\cdot\sqr{11}}+\sqr[3]{-4-i\cdot\sqr{11}}$
+++

Codice: Seleziona tutto

[tex] x__{\script 2,3}=\frac{-b\pm\sqrt{33}}{2} [/tex]
$x__{\script 2,3}=\frac{-b\pm\sqrt{33}}{2}$
+++

Codice: Seleziona tutto

[tex]L=\frac{1}{4\pi}[2\pi\sqr{4\pi^{2}+1}+\ln({2\pi+\sqr{4\pi^{2}+1})}]=3,38304428...[/tex]
$L=\frac{1}{4\pi}[2\pi\sqr{4\pi^{2}+1}+\ln({2\pi+\sqr{4\pi^{2}+1})}]=3,38304428...$
+++

Codice: Seleziona tutto

[tex]L=\frac{a}{2}[\theta\sqr{\theta^{2}+1}}+\ln({\theta+\sqr{\theta^{2}+1})}]_\theta_1^{\theta_2}[/tex]
$L=\frac{a}{2}[\theta\sqr{\theta^{2}+1}}+\ln({\theta+\sqr{\theta^{2}+1})}]_\theta_1^{\theta_2}$
+++

Codice: Seleziona tutto

[tex]\theta_1=0[/tex]
$\theta_1=0$
+++

Codice: Seleziona tutto

[tex]\theta\geq[/tex]
$\theta\geq$
+++

Codice: Seleziona tutto

[tex]\theta\leq0[/tex]
$\theta\leq0$
+++

Codice: Seleziona tutto

[tex](100 \pm 1)^{\small a} = k\/10^{\small 4}+a\/100\/(\pm 1)^{\small a-1}+(\pm 1)^{\small a}[/tex]
$(100 \pm 1)^{\small a} = k\/10^{\small 4}+a\/100\/(\pm 1)^{\small a-1}+(\pm 1)^{\small a}$
+++
Peppe

peppe
Livello 7
Livello 7
Messaggi: 906
Iscritto il: gio mag 26, 2005 2:41 pm
Località: Cirò Marina KR

Re: Ventisei

Messaggio da peppe »

Codice: Seleziona tutto

[tex]\frac{d\rho}{d\theta}[/tex]
$\frac{d\rho}{d\theta}$
+++

Codice: Seleziona tutto

[tex]\a^{\prime}[/tex]
$\a^{\prime}$
+++

Codice: Seleziona tutto

[tex]\rho^{\prime}^{2}[/tex]
$\rho^{\prime}^{2}$
+++

Codice: Seleziona tutto

[tex] \Huge {\infty} [/tex]
$\Huge {\infty}$
+++

Codice: Seleziona tutto

[tex]{\infty}[/tex]
${\infty}$
+++

Codice: Seleziona tutto

[tex]\lim_{x\to\infty}[/tex]
$\lim_{x\to\infty}$
+++

Codice: Seleziona tutto

[tex]\lim_{x\to\infty} S_n[/tex] = [tex]\frac{a_1}{1-q}[/tex]
$\lim_{x\to\infty} S_n$ = $\frac{a_1}{1-q}$
+++

Codice: Seleziona tutto

[tex]L=\displaystyle\int_{\theta_1}^{\theta_2}\sqr{{\rho}^{2}(\theta)+\rho^{\prime}^{2}(\theta)}\ d\theta[/tex]
$L=\displaystyle\int_{\theta_1}^{\theta_2}\sqr{{\rho}^{2}(\theta)+\rho^{\prime}^{2}(\theta)}\ d\theta$
+++

Codice: Seleziona tutto

[tex]L=\displaystyle\int_{\theta_1}^{\theta_2}\sqr{{(a\theta)}^{2}+a^{2}}\ d\theta=a\displaystyle\int_{\theta_1}^{\theta_2}\sqr{{\theta}^{2}+1}\ d\theta[/tex]
$L=\displaystyle\int_{\theta_1}^{\theta_2}\sqr{{(a\theta)}^{2}+a^{2}}\ d\theta=a\displaystyle\int_{\theta_1}^{\theta_2}\sqr{{\theta}^{2}+1}\ d\theta$
+++

Codice: Seleziona tutto

[tex]\frac{x}{y} =  \sum_{i=0}^{n} \frac{(B*M^{i})}{10^{P(i+1)}[/tex]
$\frac{x}{y} = \sum_{i=0}^{n} \frac{(B*M^{i})}{10^{P(i+1)}$
+++
Peppe

peppe
Livello 7
Livello 7
Messaggi: 906
Iscritto il: gio mag 26, 2005 2:41 pm
Località: Cirò Marina KR

Re: Ventisei

Messaggio da peppe »

Codice: Seleziona tutto

[tex](\frac{a}{b}+\sqrt{a^2+b^2}) \Large \sum_{i=1}^n(\frac{1}{i})[/tex]
$(\frac{a}{b}+\sqrt{a^2+b^2}) \Large \sum_{i=1}^n(\frac{1}{i})$
+++

Codice: Seleziona tutto

[tex] \frac{1} {6} \sum_{\small{i=0}}^{\infty}(\frac{5} {6})^{\small{2i}} [/tex]
$\frac{1} {6} \sum_{\small{i=0}}^{\infty}(\frac{5} {6})^{\small{2i}}$
+++

Codice: Seleziona tutto

[tex]s^{\script \prime} = \sum {\left( 4 - i \right) \, n_{\script i} } = 4m - h[/tex]
$s^{\script \prime} = \sum {\left( 4 - i \right) \, n_{\script i} } = 4m - h$
+++

Codice: Seleziona tutto

[tex]\begin{pmatrix}a & a_p\\b & b_p\\ \end{pmatrix}=\begin{pmatrix}9 & 6\\11 & 16\\ \end{pmatrix}[/tex]
$\begin{pmatrix}a & a_p\\b & b_p\\ \end{pmatrix}=\begin{pmatrix}9 & 6\\11 & 16\\ \end{pmatrix}$
+++

Codice: Seleziona tutto

[tex]\begin{pmatrix}1 & 2 & 3\\4 & 5 & 6\\ \end{pmatrix}[/tex]
$\begin{pmatrix}1 & 2 & 3\\4 & 5 & 6\\ \end{pmatrix}$
+++

Codice: Seleziona tutto

[tex]\begin{pmatrix}1 & 2 & 3\\4 & 5 & 6\\ \end{pmatrix}[/tex]
$\begin{pmatrix}1 & 2 & 3\\4 & 5 & 6\\ \end{pmatrix}$
+++

Codice: Seleziona tutto

[tex]
  \left(
    \begin{array}{ccc}
      1 & 2 & 3\\
      4 & 5 & 6
    \end{array}
  \right)
[/tex]
$\left( \begin{array}{ccc} 1 & 2 & 3\\ 4 & 5 & 6 \end{array} \right)$
Peppe

peppe
Livello 7
Livello 7
Messaggi: 906
Iscritto il: gio mag 26, 2005 2:41 pm
Località: Cirò Marina KR

Re: Ventisei

Messaggio da peppe »

Codice: Seleziona tutto

[tex]\begin{pmatrix}1 & 2 & 3 & 4 & 5 & 6 & 7 & 8 & 9 & 10\\11 & 12 & 13 & 14 & 15 & 16 & 17 & 18 & 19 & 20\\21 & 22 & 23 & 24 & 25 & 26 & 27 & 28 & 29 & 30\\31 & 32 & 33 & 34 & 35 & 36 & 37 & 38 & 39 & 40\\41 & 42 & 43 & 44 & 45 & 46 & 47 & 48 & 49 & 50\\51 & 52 & 53 & 54 & 55 & 56 & 57 & 58 & 59 & 60\\61 & 62 & 63 & 64 & 65 & 66 & 67 & 68 & 69 & 70\\71 & 72 & 73 & 74 & 75 & 76 & 77 & 78 & 79 & 80\\81 & 82 & 83 & 84 & 85 & 86 & 87 & 88 & 89 & 90\\91 & 92 & 93 & 94 & 95 & 96 & 97 & 98 & 99 & 100\\ \end{pmatrix}[/tex]
$\begin{pmatrix}1 & 2 & 3 & 4 & 5 & 6 & 7 & 8 & 9 & 10\\11 & 12 & 13 & 14 & 15 & 16 & 17 & 18 & 19 & 20\\21 & 22 & 23 & 24 & 25 & 26 & 27 & 28 & 29 & 30\\31 & 32 & 33 & 34 & 35 & 36 & 37 & 38 & 39 & 40\\41 & 42 & 43 & 44 & 45 & 46 & 47 & 48 & 49 & 50\\51 & 52 & 53 & 54 & 55 & 56 & 57 & 58 & 59 & 60\\61 & 62 & 63 & 64 & 65 & 66 & 67 & 68 & 69 & 70\\71 & 72 & 73 & 74 & 75 & 76 & 77 & 78 & 79 & 80\\81 & 82 & 83 & 84 & 85 & 86 & 87 & 88 & 89 & 90\\91 & 92 & 93 & 94 & 95 & 96 & 97 & 98 & 99 & 100\\ \end{pmatrix}$
+++

Codice: Seleziona tutto

[tex]\left(\begin{array}{cccccccccc}1 & 2 & 3 & 4 & 5 & 6 & 7 & 8 & 9 & 10\\11 & 12 & 13 & 14 & 15 & 16 & 17 & 18 & 19 & 20\\21 & 22 & 23 & 24 & 25 & 26 & 27 & 28 & 29 & 30\\31 & 32 & 33 & 34 & 35 & 36 & 37 & 38 & 39 & 40\\41 & 42 & 43 & 44 & 45 & 46 & 47 & 48 & 49 & 50\\51 & 52 & 53 & 54 & 55 & 56 & 57 & 58 & 59 & 60\\61 & 62 & 63 & 64 & 65 & 66 & 67 & 68 & 69 & 70\\71 & 72 & 73 & 74 & 75 & 76 & 77 & 78 & 79 & 80\\81 & 82 & 83 & 84 & 85 & 86 & 87 & 88 & 89 & 90\\91 & 92 & 93 & 94 & 95 & 96 & 97 & 98 & 99 & 100\end{array}\right)[/tex]
$\left(\begin{array}{cccccccccc}1 & 2 & 3 & 4 & 5 & 6 & 7 & 8 & 9 & 10\\11 & 12 & 13 & 14 & 15 & 16 & 17 & 18 & 19 & 20\\21 & 22 & 23 & 24 & 25 & 26 & 27 & 28 & 29 & 30\\31 & 32 & 33 & 34 & 35 & 36 & 37 & 38 & 39 & 40\\41 & 42 & 43 & 44 & 45 & 46 & 47 & 48 & 49 & 50\\51 & 52 & 53 & 54 & 55 & 56 & 57 & 58 & 59 & 60\\61 & 62 & 63 & 64 & 65 & 66 & 67 & 68 & 69 & 70\\71 & 72 & 73 & 74 & 75 & 76 & 77 & 78 & 79 & 80\\81 & 82 & 83 & 84 & 85 & 86 & 87 & 88 & 89 & 90\\91 & 92 & 93 & 94 & 95 & 96 & 97 & 98 & 99 & 100\end{array}\right)$
+++

Codice: Seleziona tutto

[tex]
  \left(
    \begin{array}{cccccccccc}
      1 & 2 & 3 & 4 & 5 & 6 & 7 & 8 & 9 & 10\\
      11 & 12 & 13 & 14 & 15 & 16 & 17 & 18 & 19 & 20\\
      21 & 22 & 23 & 24 & 25 & 26 & 27 & 28 & 29 & 30\\
      31 & 32 & 33 & 34 & 35 & 36 & 37 & 38 & 39 & 40\\
      41 & 42 & 43 & 44 & 45 & 46 & 47 & 48 & 49 & 50\\
      51 & 52 & 53 & 54 & 55 & 56 & 57 & 58 & 59 & 60\\
      61 & 62 & 63 & 64 & 65 & 66 & 67 & 68 & 69 & 70\\
      71 & 72 & 73 & 74 & 75 & 76 & 77 & 78 & 79 & 80\\
      81 & 82 & 83 & 84 & 85 & 86 & 87 & 88 & 89 & 90\\
      91 & 92 & 93 & 94 & 95 & 96 & 97 & 98 & 99 & 100
    \end{array}
  \right)
[/tex]
$\left( \begin{array}{cccccccccc} 1 & 2 & 3 & 4 & 5 & 6 & 7 & 8 & 9 & 10\\ 11 & 12 & 13 & 14 & 15 & 16 & 17 & 18 & 19 & 20\\ 21 & 22 & 23 & 24 & 25 & 26 & 27 & 28 & 29 & 30\\ 31 & 32 & 33 & 34 & 35 & 36 & 37 & 38 & 39 & 40\\ 41 & 42 & 43 & 44 & 45 & 46 & 47 & 48 & 49 & 50\\ 51 & 52 & 53 & 54 & 55 & 56 & 57 & 58 & 59 & 60\\ 61 & 62 & 63 & 64 & 65 & 66 & 67 & 68 & 69 & 70\\ 71 & 72 & 73 & 74 & 75 & 76 & 77 & 78 & 79 & 80\\ 81 & 82 & 83 & 84 & 85 & 86 & 87 & 88 & 89 & 90\\ 91 & 92 & 93 & 94 & 95 & 96 & 97 & 98 & 99 & 100 \end{array} \right)$
+++

Codice: Seleziona tutto

[tex]A_1=\pm\frac{1}{2}\begin{vmatrix}\frac{5}{14} & \frac{\sqr{3}}{14} & 1\\\frac{3}{7} & \frac{2\sqr{3}}{7} & 1\\\frac{5}{7} & \frac{\sqr{3}}{7} & 1\end{vmatrix}=\frac{\sqr{3}}{28}[/tex]
$A_1=\pm\frac{1}{2}\begin{vmatrix}\frac{5}{14} & \frac{\sqr{3}}{14} & 1\\\frac{3}{7} & \frac{2\sqr{3}}{7} & 1\\\frac{5}{7} & \frac{\sqr{3}}{7} & 1\end{vmatrix}=\frac{\sqr{3}}{28}$
+++
Peppe

peppe
Livello 7
Livello 7
Messaggi: 906
Iscritto il: gio mag 26, 2005 2:41 pm
Località: Cirò Marina KR

Re: Ventisei

Messaggio da peppe »

Codice: Seleziona tutto

[tex]\begin{pmatrix}ab_p & a_pb\\b & b_p\\ \end{pmatrix}=\begin{pmatrix}144 & 66\\11 & 16\\ \end{pmatrix}[/tex]
$\begin{pmatrix}ab_p & a_pb\\b & b_p\\ \end{pmatrix}=\begin{pmatrix}144 & 66\\11 & 16\\ \end{pmatrix}$
+++

Codice: Seleziona tutto

[tex]\begin{pmatrix}ab_p + a_pb\\b + b_p\\ \end{pmatrix}=\begin{pmatrix}210\\27\\ \end{pmatrix}[/tex]
$\begin{pmatrix}ab_p + a_pb\\b + b_p\\ \end{pmatrix}=\begin{pmatrix}210\\27\\ \end{pmatrix}$
+++

Codice: Seleziona tutto

[tex]A_1=\pm\frac{1}{2}\begin{vmatrix}
\frac{5}{14} & \frac{\sqr{3}}{14} & 1\\\frac{3}{7} & \frac{2\sqr{3}}{7} & 1\\\frac{5}{7} & \frac{\sqr{3}}{7} & 1\end{vmatrix}=\frac{\sqr{3}}{28}[/tex]
$A_1=\pm\frac{1}{2}\begin{vmatrix} \frac{5}{14} & \frac{\sqr{3}}{14} & 1\\\frac{3}{7} & \frac{2\sqr{3}}{7} & 1\\\frac{5}{7} & \frac{\sqr{3}}{7} & 1\end{vmatrix}=\frac{\sqr{3}}{28}$
+++

Codice: Seleziona tutto

[tex]\quad a=b\\2)\quad b=c\\3)\\4)\\5)[/tex]
$\quad a=b\\2)\quad b=c\\3)\\4)\\5)$
+++

Codice: Seleziona tutto

[tex]\delta _{\script ij} = \left \{ {\begin{array}{cc} 1 & {i = j} \\ 0 & {i \ne j} \\ \end{array}} \right.[/tex]
$\delta _{\script ij} = \left \{ {\begin{array}{cc} 1 & {i = j} \\ 0 & {i \ne j} \\ \end{array}} \right.$
+++

Codice: Seleziona tutto

[tex]p \left( h | k \, m \, I \right) = \frac {C \left( B \right) C \left( S \right)} C = \frac {{{4m + k} \choose {k - h}} {{4m + k} \choose {k + h}}} {{{8m + 2k} \choose {2k}}}[/tex]
$p \left( h | k \, m \, I \right) = \frac {C \left( B \right) C \left( S \right)} C = \frac {{{4m + k} \choose {k - h}} {{4m + k} \choose {k + h}}} {{{8m + 2k} \choose {2k}}}$
+++

Codice: Seleziona tutto

[tex]C = \underbrace {{{8m} \choose 4} {{8m - 4} \choose 4} \cdots {4 \choose 4}}_{2m} = \frac {\left( 8m \right)!} {4!^{\script 2m}}[/tex]
$C = \underbrace {{{8m} \choose 4} {{8m - 4} \choose 4} \cdots {4 \choose 4}}_{2m} = \frac {\left( 8m \right)!} {4!^{\script 2m}}$
+++

Codice: Seleziona tutto

[tex]\left \{ x_{\script 1}^{\script 2} + y_{\script 1}^{\script 2} + \alpha x_{\script 1} + \beta y_{\script 1} +\gamma =0 \\ x_{\script 2}^{\script 2} + y_{\script 2}^{\script 2} + \alpha x_{\script 2} + \beta y_{\script 2} +\gamma =0 \\ x_{\script 3}^{\script 2} + y_{\script 3}^{\script 2} + \alpha x_{\script 3} + \beta y_{\script 3} +\gamma =0 \right .[/tex]
$\left \{ x_{\script 1}^{\script 2} + y_{\script 1}^{\script 2} + \alpha x_{\script 1} + \beta y_{\script 1} +\gamma =0 \\ x_{\script 2}^{\script 2} + y_{\script 2}^{\script 2} + \alpha x_{\script 2} + \beta y_{\script 2} +\gamma =0 \\ x_{\script 3}^{\script 2} + y_{\script 3}^{\script 2} + \alpha x_{\script 3} + \beta y_{\script 3} +\gamma =0 \right .$
+++

Codice: Seleziona tutto

[tex]\reverse\opaque \frac {MDC(a^{\small 2}, b^{\small 2})} {MDC(ab, a+b)} = \frac {m^{\small 2}} {p} = mq[/tex]
$\reverse\opaque \frac {MDC(a^{\small 2}, b^{\small 2})} {MDC(ab, a+b)} = \frac {m^{\small 2}} {p} = mq$
+++

Codice: Seleziona tutto

[tex]\reverse\opaque \frac {2}{32}[/tex]
$\reverse\opaque \frac {2}{32}$
++++
E per oggi basta!!!
---
Per Pietro:se il post occupa troppo spazio dimmelo così lo cancello.ciao peppe
Peppe

Pasquale
Livello 12
Livello 12
Messaggi: 2854
Iscritto il: mer mag 25, 2005 2:14 am

Re: Ventisei

Messaggio da Pasquale »

La dimostrazione viene fatta capovolgendo la domanda: considerato che 27-25=2 ed in mezzo c'è il 26, allora dati due numeri x ed y, con x<y, trovare tutti gli x ed y tali che:

y^3-x^2=2

Risolvendo l'equazione, si vede che ha una sola soluzione: x=25; y=27.

Il procedimento, per me troppo sintetico, non l'ho capito bene, ma il concetto è questo.
_________________

$\text { }$ciao Immagine ciao
E' la somma che fa il totale (Totò)

fabtor
Livello 5
Livello 5
Messaggi: 226
Iscritto il: mar nov 17, 2009 3:59 pm

Re: Ventisei

Messaggio da fabtor »

Secondo me ci si sta un po' complicando la vita!

Premetto che non so come ci sia arrivato Fermat ma io farei così:

Poichè sto cercando un numero a cavallo tra un quadrato ed un cubo questo implica che il primo differisce dal secondo di 2 unità per cui scrivo: y^2 + 2 = z^3

A questo punto farei queste considerazioni:

A sx dell'uguale abbiamo una funzione sempre positiva (è una parabola con minimo 2) monotona crescente (e a maggior ragione per Y appartenente ad |N).

A dx dell'uguale abbiamo una funzione sempre positiva per z appartenente ad N con Z > 0 anch'essa monotona crescente.

Si osserva inoltre che Y^2 + 2 cresce come Y^2 e lo fa più lentamente di Z^3

Quindi abbiamo due casi possibili o Y^2 + 2 non incontra mai Z^3 in |R, e ciò si dimostra agevolmente che accade per Y < Z
o se la incontra lo fa in un unico punto in |R, e ciò accade per Y > Z (se Y = Z avremmo y^2 + 2 = y^3 che non ha soluzioni per Y e Z appartenenti a |N).

Tuttavia noi sappiamo già (come del resto Fermat stesso presumo) che una soluzione di questa equazione è 25 +2 =27
cioè y = 5 e z = 3 quindi 26 è l'unico numero in |R che soddisfa l'equazione ed appartenendo a |N a maggior ragione 26 è l'unico numero a cavallo tra un quadrato (5^2 = 25) e un cubo (3^3 = 27).

N.B. Si osserva che effettivamente Y > Z come asseriva "la teoria".

P.S. In parallelo sto cercando di trovare anche una dimostrazione più analitica, ma al momento sono solo riuscito a dimostrare che se Y = 5 e Z = 3 non sono le uniche soluzioni le altre possibili se esistono sono con Y,Z dispari e ponendo Z = 2g + 1, g è a sua volta dispari.
Ah, se i portieri avessero sulla maglia: $|e^{-i\pi}|$...

Pongo $y = x^{2}$ quindi $y=\frac {x^{2}}{pongo}$
[tratto da un compito in classe di uno studente di prima superiore]

Il vero gnomone aureo: http://thumbs.dreamstime.com/z/gnomo-de ... 526933.jpg

fabtor
Livello 5
Livello 5
Messaggi: 226
Iscritto il: mar nov 17, 2009 3:59 pm

Re: Ventisei

Messaggio da fabtor »

Riporto la Dimostrazione "più analitica" (completata) che ho ricavato in base ad alcune osservazioni.

Se x è a cavallo tra un quadrato ed un cubo ciò implica che ci troviamo nel caso:

(x-1)^(1/2) appartiene ad N e (x+1)^(1/3) appartiene a N

Ponendo (x-1)^(1/2) = y e (x+1)^(1/3) = z si ha che:

y^2 + 2 = Z^3 con y,z appartenenti a N

Cioè y^2 + 2 -27 = Z^3 -27 da cui (y + 5)(y - 5) = (z - 3)(z^2 + 3z + 9)

Si osserva che per sussistere l'ugualianza tra i due termini a dx e a sx dell'uguale questi devono avere concordanza di segno

e che il termine a dx dell'uguale è negativo per z < 3 e positivo o nullo per z>=3
mentre il termine a sx è positivo per y <= -5 e y >= 5 e negativo per -5 <= y <= 5.

Poichè y,z appartengono a N si ha che

il termine a dx dell'uguale è negativo per 0 <= z < 3 e positivo o nullo per z>=3
il termine a sx è positivo per y >= 5 e negativo per 0 <= y <= 5.

Essendo y^2 + 2 = Z^3 e y appartine a N allora y = (z^3-2)^(1/2) con C.E. z >= (2)^(1/3)

Abbiamo quindi 2 casi :

0 <= z < 3 e z >= (2)^(1/3) con z Appartenente a N cioè z = 2 (termine a dx < 0)

z>=3 e z >= (2)^(1/3) con z Appartenente a N cioè z >= 3 (termine a dx >=0)

Per la concordanza se termine a dx < 0 allora termine a sx < 0

quindi si ha che per z = 2: y = |(6)^(1/2)| che però non appartine a N quindi deve essere Z<>2 cioè

I termini a sx e a dx dell'uguale devono essere > 0 cioè y>=5 e z>=3 con y,z appartenenti a N

Riassumendo abbiamo il sistema :

y^2 + 2 = z ^3
y >= 5
z >= 3

Con tre casi possibili:

(1) y = z
(2) y < z ( che implica che y^2 + 2 < z^2 + 2 con Z^3 < z^2 + 2)
(3) y > z

Svolgendo i conti ai osserva che:

la (1) implica che 1 < y < 2 cioè y non appartenente ad N che quindi contraddice l'ipotesi
la (2) implica che z < 6, ma ciò è assurdo poichè essendo z > y, z appartenente a N e y>=5 deve essere z>=6

Quindi l'unico caso possibile resta y > z

Cioè il sistema diventa composto da:

y^2 + 2 = z ^3
y > z
y >= 5
z >= 3

Osserviamo che per i minimi di y e z otteniamo proprio x = 26
e che inoltre in questo caso si ha che y - z = z^3 - y^2 che implica:

y = z + 2 --> y^2 = (z + 2)^2
y^2 = z^3 -2

cioè = z^3 - z^2 - 4z -6 = 0 che ha soluzioni reali solo per z = 3

Quindi se y-z =z^3 -y^2 è una condizione necessaria affinchè y, x, z appartengano a N con x: (x-1)^2 + 2 = (x+1)^3
allora si ha che l'unico valore di x è 26.

Verifichiamo dunque che tale condizione è effettivamente necessaria.

Abbiamo due casi alternativi:
y-z < z^3 -y^2 (A)
y-z > z^3 -y^2 (B)
La (A) implica che z<3 quindi si avrebbe il sistema formato da

z >= 3
z < 3
che è impossibile

La (B) implica invece che z>3 quindi si avrebbe il sistema formato da

z >= 3
z > 3

Cioè z > 3 con z appartenente a N --> z>=4 che implica che y< -(62)^(1/2) e y> (62)^(1/2) che con y appartenente a N
Implica che y >=8

Cioè per y-z >2 (cioè per y-z= 2k con k >=2 e k appartenente a N) si ha il sistema:

y^2 + 2 = z ^3 (c)
y-z >2 (y > z)
y >= 8
z >= 4

Si osserva inoltre che se z^3 è pari anche z e y devono essere pari poichè l'ugualianza (c) sussista
e che se z^3 è dispari anche z e y devono essere dispari poichè l'ugualianza (c) sussista

Quindi z e y devono avere la stessa parità.

Se z e y sono entrambi pari allora z =2g e y = 2h e sostituendo in (c) si ottiene che 2g^3 - h^2 = 1/2, ma ciò è assurdo poichè g e h appartengono a N e quindi a maggior ragione anche 2g^3 - h^2 deve essere almeno intero.
Quindi y e z devono essere dispari.

Il sistema diventa quindi:

y^2 + 2 = z ^3 (c)
y-z =2k con k>=2 e k appartenente a N (y > z)
y >= 9 (con y solo dispari)
z >= 5 (con z solo dispari)

se z e y sono dispari ponendo y= 2h +1 e z= 2g +1 sostituendo in (c) si ottiene:

2h^2 +2h +1 = g(4g^2+ 6g +1) che poniamo nella forma I = II

Si osserva che I è dispari per qualunque h
Si osserva che II è dispari per g dispari

Quindi affinchè l'ugualianza sussista g deve essere dispari.
Pongo quindi g = 2m + 1 con m appartenente ad N e sostituendo la posizione fatta nella formula I = II

ottengo 2h^2 +2h +1 = (2m+1)^2 X (2m +13) che ripongo nella forma I = II

A questo punto si osserva che I è indecomponibile in R e che quindi per sussistere l'ugualianza almeno uno dei due fattori di II deve essere pari 1.
Si osserva che se 2m + 13 = 1 allora m =-6 --> z= -21 non appartenente a N e tale soluzione non è accettabile.
Si osserva che se 2m + 1 = 1 allora m = 0 e quindi 2h^2 +2h +1 = 13 cioè

h(1) = -3 h(2) = 2, poichè h deve appartenere a N
l'unica coppia di valori accettabili sono h = 2 e m = 0 ma tali valori implicano che y= 5 e z = 3, ma le nostre condizioni implicano che y>= 9 e z>=5 quindi le soluzioni trovate per y-z>2 non sono accettabili.

Riassumendo
abbiamo dimostrato che
y-z < 2 da soluzioni impossibili
y-z > 2 da soluzioni non accettabili
y-z = 2 da come unica soluzione y= 5, z= 3

Quindi le uniche soluzioni che soddisfano y^2 + 2 = z ^3 sono quelle per y - z = 2 che implicano secondo la posizione iniziale (x-1)^(1/2) = y che x -1 =5^2 --> x-1 = 25 --> x = 26 (la si ottiene analogamente anche con (x+1)^(1/3) = z
Quindi 26 è effettivamente l'unico numero intero a cavallo tra un quadrato ed un cubo.
C.V.D.

Beh, questo è quanto: non credo di aver commesso degli errori nel mio ragionamento, ma in caso contrario vi chiedo scusa per il tempo che vi ho fatto perdere nella lettura.
Ah, se i portieri avessero sulla maglia: $|e^{-i\pi}|$...

Pongo $y = x^{2}$ quindi $y=\frac {x^{2}}{pongo}$
[tratto da un compito in classe di uno studente di prima superiore]

Il vero gnomone aureo: http://thumbs.dreamstime.com/z/gnomo-de ... 526933.jpg

Tino
Livello 5
Livello 5
Messaggi: 221
Iscritto il: mer mag 25, 2005 9:20 am
Località: Verona

Re: Ventisei

Messaggio da Tino »

Ciao a tutti! :D

Caro fabtor, purtroppo entrambe le tue soluzioni sono sbagliate :)
fabtor ha scritto:Quindi abbiamo due casi possibili o Y^2 + 2 non incontra mai Z^3 in |R, e ciò si dimostra agevolmente che accade per Y < Z
o se la incontra lo fa in un unico punto in |R, e ciò accade per Y > Z (se Y = Z avremmo y^2 + 2 = y^3 che non ha soluzioni per Y e Z appartenenti a |N).
Non in un unico punto. Te ne so trovare almeno un altro: $y=1$, $z=\sqrt[3]{3}$. Ma ce ne sono ovviamente infiniti.
fabtor ha scritto:ottengo 2h^2 +2h +1 = (2m+1)^2 X (2m +13) che ripongo nella forma I = II

A questo punto si osserva che I è indecomponibile in R e che quindi per sussistere l'ugualianza almeno uno dei due fattori di II deve essere pari 1.
Che quell'espressione sia indecomponibile non significa che non esistano $h,m$ interi che la verificano. Per esempio $x^2+5= (y-1)(y-2)$ per $x=5$ e $y=7$, eppure $x^2+5$ e' indecomponibile.
"Oh! But I have been blind- blind. Complex, I have said?
Complicated? Mais non. Of a simplicity extreme - extreme.
And miserable one that I am, I saw nothing - nothing."
(Peril At End House)

Tino
Livello 5
Livello 5
Messaggi: 221
Iscritto il: mer mag 25, 2005 9:20 am
Località: Verona

Re: Ventisei

Messaggio da Tino »

Vorrei aggiungere la dimostrazione "algebrica classica" del fatto che le uniche soluzioni intere di $x^2+2=y^3$ sono $(x,y) = (\pm 5,3)$.

Il fatto non elementare che serve e' il seguente: la fattorizzazione in irriducibili nell'anello $A=\mathbb{Z}[\sqrt{-2}] \subset \mathbb{C}$ (cioe' l'insieme degli elementi di $\mathbb{C}$ della forma $\alpha + i \sqrt{2} \beta$ con $\alpha, \beta \in \mathbb{Z}$) e' unica.

Niente di cui spaventarsi, succede come in $\mathbb{Z}$, in altre parole :mrgreen:

Qui sto usando la notazione $i=\sqrt{-1} \in \mathbb{C}$.

Ora riscriviamo $x^2+2 = y^3$ come $(x+\sqrt{-2})(x-\sqrt{-2}) = y^3$. Se mostriamo che $x+\sqrt{-2}$ e $x-\sqrt{-2}$ sono coprimi in $A$ - cioe' non hanno fattori comuni non banali in $A$ - allora per la fattorizzazione unica potremo dedurre che $x+\sqrt{-2}$ e' un cubo, cioe' della forma $(m+n\sqrt{-2})^3$ con $m+n \sqrt{-2} \in A$ (cioe' $m,n \in \mathbb{Z}$). Questo ci piacera' molto.

Prendiamo un elemento $a \in A$ che divide $x+\sqrt{-2}$ e $x-\sqrt{-2}$. Allora $a$ divide $(x+\sqrt{-2}) - (x-\sqrt{-2}) = 2 \sqrt{-2}$, quindi la sua "norma" (nel senso complesso: lui moltiplicato il suo coniugato) deve dividere la norma di $2 \sqrt{-2}$, cioe' $8$. In particolare la norma di $a$ e' $1$ oppure un intero pari.

D'altra parte la norma di $a$ deve dividere la norma di $y^3$, che e' un intero dispari! Infatti $y$ stesso e' dispari (se $y$ fosse pari allora siccome $x^2+2=y^3$ si avrebbe che $x^2 \equiv 2$ modulo $4$, impossibile).

Quindi $a$ ha norma $1$, cioe' $a=1$ oppure $a=-1$ (ricordo che $a$ e' della forma $\alpha + \beta \sqrt{-2}$).

Segue che possiamo scrivere $x+\sqrt{-2} = (m+n\sqrt{-2})^3$, e questo dopo un po' di conti ci porta a concludere che $x=m^3-6mn^2$ e $1=-2n^3+3m^2n$. Da quest'ultima equazione segue che $n = 1$ e $m=\pm 1$. Dalla prima $x = \pm 5$, da cui $y=3$.
Ultima modifica di Tino il sab mag 07, 2011 1:10 am, modificato 1 volta in totale.
"Oh! But I have been blind- blind. Complex, I have said?
Complicated? Mais non. Of a simplicity extreme - extreme.
And miserable one that I am, I saw nothing - nothing."
(Peril At End House)

fabtor
Livello 5
Livello 5
Messaggi: 226
Iscritto il: mar nov 17, 2009 3:59 pm

Re: Ventisei

Messaggio da fabtor »

Accidenti Tino, ha ragione in entrambi i casi (anche se credo che al massimo la ns cubica ed la ns parabola possano arrivare ad incrociarsi in 4 punti), tuttavia credo che ragionandoci un po' su (ed appena avrò un po' di tempo mi riprometto di farlo almeno la prima delle 2 (ma forse anche la seconda) possa essere corretta...
Grazie inoltre per la dimostrazione "aritmetica classica" anche se mi sento di dubitare che fosse quella di Fermat.
Ah, se i portieri avessero sulla maglia: $|e^{-i\pi}|$...

Pongo $y = x^{2}$ quindi $y=\frac {x^{2}}{pongo}$
[tratto da un compito in classe di uno studente di prima superiore]

Il vero gnomone aureo: http://thumbs.dreamstime.com/z/gnomo-de ... 526933.jpg

Tino
Livello 5
Livello 5
Messaggi: 221
Iscritto il: mer mag 25, 2005 9:20 am
Località: Verona

Re: Ventisei

Messaggio da Tino »

fabtor ha scritto:credo che al massimo la ns cubica ed la ns parabola possano arrivare ad incrociarsi in 4 punti
Com'e' possibile? :) per ogni $x$ fissato se definiamo $z=(x^2+2)^{1/3}$ otteniamo $x^2+2=z^3$. Quindi le soluzioni reali sono infinite.
fabtor ha scritto:Grazie inoltre per la dimostrazione "aritmetica classica" anche se mi sento di dubitare che fosse quella di Fermat.
Sono d'accordo, anche perche' (non vorrei dire una fesseria) non credo che allora ci fosse la nozione di anello.
"Oh! But I have been blind- blind. Complex, I have said?
Complicated? Mais non. Of a simplicity extreme - extreme.
And miserable one that I am, I saw nothing - nothing."
(Peril At End House)

Rispondi